Oi pessoal,estou sem ideias para este problema:

Considere um número real α e constantes b > 0 e γ ≥ 1 tais que para
quaisquer p e q inteiros com q ≥ 1 vale
|qα − p| ≥ b/qγ.
Prove que existe uma constante C tal que, para todo inteiro N ≥ 1, o
conjunto

XN = {mα − ɭmα⌡, m ∈ Z, 0 ≤ m ≤ CNγ}
é tal que, para todo x ∈ [0, 1] existe y ∈ XN com |x − y| < 1/N.

nota: ɭmα⌡ é a parte inteira de mα.

Alguem tem alguma sugestao de como desenvolver uma bom raciocinio para ela?
Como voces a atacariam?

Abraços

-- 
Esta mensagem foi verificada pelo sistema de antiv�rus e
 acredita-se estar livre de perigo.

Responder a